[PDF] [PDF] Week 3 Solutions Page 1 Exercise (241) Prove that ) is Cauchy

(n2−1 n2 ) is Cauchy using directly the definition of Cauchy sequences Proof Given ϵ > 0 Let {xn} be a sequence such that there exists a 0



Previous PDF Next PDF





[PDF] Week 3 Solutions Page 1 Exercise (241) Prove that ) is Cauchy

(n2−1 n2 ) is Cauchy using directly the definition of Cauchy sequences Proof Given ϵ > 0 Let {xn} be a sequence such that there exists a 0



[PDF] Midterm Solutions

Let X = (xn) be a sequence of positive real numbers such that lim (xn+1 xn ) is Cauchy 4 Let (fn) ∈ C[0,1] be such that there exists M > 0 such that fn ∞ ≤



[PDF] MATH 403 ANALYSIS I - SPRING 2010 SOLUTIONS to

A real sequence (xn) is called contractive if there exists a constant 0 0 and let N be such that x2 − x1 Cn−1 1 1−C



[PDF] Practice Problems 3 : Cauchy criterion, Subsequence

(c) If (xn) satisfies the Cauchy criterion, then there exists an α ∈ R such that 0



[PDF] Let xn be a monotone increasing sequence bounded above and con

every monotone increasing bounded sequence bounded above converges that for every ε > 0 there exists some n > 1/ε such that xn < ε 1 (c) Using the Archimedean property, argue that yk cannot be bounded above by M, hence



[PDF] MA 101 (Mathematics I) Hints/Solutions for Practice Problem Set - 2

If xn → 0, then there exists n0 ∈ N such that xn < 1 2 If possible, let there exist a non-convergent sequence Hence there exists c ∈ (0,∞) such that f (c) = 0



[PDF] Sequences - UC Davis Mathematics

(xn)∞ n=0 is a function f : N0 → R where xn = f(n) and N0 = {0, 1, 2, 3, }, and eventually; and (xn) does not converge to x ∈ R if there exists ϵ0 > 0 such that We let x = lim n→∞ xn, y = lim n→∞ yn The first statement is immediate if c = 0



[PDF] M17 MAT25-21 HOMEWORK 5 SOLUTIONS 1 To Hand In

(c) A divergent monotone sequence with a Cauchy subsequence Let (an) and (bn) be Cauchy sequences Decide whether each k=1 ak is Cauchy if and only if for all ϵ > 0 there exists N ∈ N such that whenever n>m ≥ N (b) A convergent series ∑xn and a bounded sequence (yn) such that ∑xnyn diverges (c) Two 



[PDF] MATH 409, Summer 2019, Practice Problem Set 2 - TAMU Math

29 mai 2019 · Show that a sequence (xn)∞ n=1 is convergent to l ∈ R, if and only if for every ε ∈ (0, 2) there exists N ∈ N such that for all n ⩾ N, xn − l < ε

[PDF] letra cancion bandolero paris latino

[PDF] letter and sound assessment form

[PDF] letter coding examples

[PDF] letter coding tricks

[PDF] letter granting permission to use copyrighted music

[PDF] letter identification assessment free

[PDF] letter identification assessment pdf

[PDF] letter of acceptance of appointment as director

[PDF] letter of appointment of additional director in private company

[PDF] letter of consent for child to travel with grandparents

[PDF] letter of consent to travel with one parent

[PDF] letter of permission to travel

[PDF] letter of permission to travel from employer

[PDF] letter pattern programs in java

[PDF] letter stating i pay rent

Week 3 Solutions Page 1

Exercise(2.4.1).Prove that

n21n 2 is Cauchy using directly the denition of

Cauchy sequences.

Proof.Given >0, letM2Nbe such thatr2

< M.

Then, for anym;nM,

jxmxnj=m21m 2n21n 2 1n 21m
2 1n 2+1m 2 1M 2+1M 2 2M 2

Therefore,

n21n 2 is a Cauchy sequence.Exercise(2.4.2).Letfxngbe a sequence such that there exists a0< C <1 such that jxn+1xnj Cjxnxn1j: Prove thatfxngis Cauchy. Hint: You can freely use the formula (forC6= 1)

1 +C+C2++Cn=1Cn+11C:

Proof.Let >0 be given. Note that

jx3x2j Cjx2x1j jx4x3j Cjx3x2j CCjx2x1j=C2jx2x1j and in general, one could prove that jxn+1xnj Cjxnxn1j C2jxn1xn2j Cn1jx2x1j:

Week 3 Solutions Page 2

Now, form > n, we can evaluate the quantity

jxmxnj jxmxm1j+jxm1xm2j++jxn+1xnj

Cm2jx2x1j+Cm3jx2x1j++Cn1jx2x1j

= (Cm2++Cn1)jx2x1j =Cn1(1 +C++Cmn1)jx2x1j =Cn11Cmn1C jx2x1j

Cn111C

jx2x1j Now, since 0< C <1,Cn1!0 asn! 1. Therefore, there existsN2N such that whenevernN, jCn10j< 11C jx2x1j:

For this sameN, wheneverm > nN

jxmxnj Cn111C jx2x1j 11C jx2x1j11C jx2x1j

Therefore,fxngis a Cauchy sequence.Exercise.Prove the following statement using Bolzano-Weierstrass theorem.

Assume that(xn)n2Nis a bounded sequence inRand that there existsx2Rsuch that any convergent subsequence(xni)i2Nconverges tox. Thenlimn!1xn=x. Proof.Assume for contradiction thatxn6!x. Then9 >0 such that jxnxj for innitely manyn. From this, we can create a subsequencefxnjg such thatjxnjxj for allj2N. Since our original sequence is bounded, this subsequence is bounded, and so, by Bolzano-Weierstrass, there is a convergent subsequence of this subsequence, fxnjkg. By assumption,fxnjkgconverges tox. However, this is a contradiction sincejxnjkxj for allk2N.

Hence, we must havexn!xasn! 1.

Week 3 Solutions Page 3

Exercise.Show that

a) the setZ=f:::;1;0;1;:::ghas no cluster points. b) every point inRis a cluster point ofQ. Proof.a) Ifx2Z, then (x1=2;x+1=2)\Znfxg=;and soxis not a cluster point ofZ. Ifx62Z, then9k2Zsuch thatx2(k;k+ 1). Choose= minfjxkj;jx (k+1)jg. Then (x;x+)\Znfxg=;, and soxis not a cluster point of Z.

ThereforeZhas no cluster points.

b) Letx2R. Let >0. By the density ofQ,9r2Qsuch thatx < r < x+. Then (x;x+)\Qnfxg 6=;. Sincewas arbitrary, this shows thatxis a cluster point ofQ. Sincexwas arbitrary, every point inRis a cluster point ofQ.Exercise.In the lecture we have shown any Cauchy sequence(xn)n2NRhas a limit inR, i.e. there existsx2Rwithlimn!1xn=x.(1) The same statement is false inQ, the following is false: any Cauchy sequence(xn)n2NQhas a limit inQ, i.e. there existsx2Qwithlimn!1xn=x.(2) a) Give a counterexample to (2). b) Which part of the proof of (1) (from the lecture) fails when we attempt to prove (2)? Proof.a) Dene a sequencefxngas follows. For alln2N, choosexn2Qsuch thatp21=n < xnWeek 3 Solutions Page 4quotesdbs_dbs6.pdfusesText_12